LSAT and Law School Admissions Forum

Get expert LSAT preparation and law school admissions advice from PowerScore Test Preparation.

User avatar
 Dave Killoran
PowerScore Staff
  • PowerScore Staff
  • Posts: 5852
  • Joined: Mar 25, 2011
|
#71216
This game is discussed in our Podcast: LSAT Podcast Episode 31: The September 2019 LSAT Logic Games Section


Complete Question Explanation
(The complete setup for this game can be found here: https://forum.powerscore.com/lsat/viewtopic.php?t=31306)

The correct answer choice is (C).

If S is in case 3, we can use the hypothetical created at the end of our setup discussion to perhaps eliminate a few answers. In this case, we'll modify the hypo by placing S in case 3:

  • J/P ..... G/N ..... _S_ ..... N/G ..... P/J ..... _M_ ..... _H_
     1 .....          2 .....    3 .....       4 .....      5 .....      6 .....      7

Answer choice (A): Our hypothetical shows that N could also be in case 4, and so this answer choice is incorrect.

Answer choice (B): Our hypothetical shows that P could also be in case 1, and so this answer choice is incorrect.

Answer choice (C): This is the correct answer choice. The hypothetical above by itself doesn't prove M must be in case 6, but since it eliminates every other answer, it turns out that it must in fact be the case. Single hypotheticals aren't usually this powerful, but in the instance it worked out in our favor.

Answer choice (D): Our hypothetical shows that G could also be in case 2, and so this answer choice is incorrect.

Answer choice (E): Our hypothetical shows that J could also be in case 5, and so this answer choice is incorrect.

Get the most out of your LSAT Prep Plus subscription.

Analyze and track your performance with our Testing and Analytics Package.